Download Chapter 1 - Screenshot for timg.co.il

Survey
yes no Was this document useful for you?
   Thank you for your participation!

* Your assessment is very important for improving the workof artificial intelligence, which forms the content of this project

Document related concepts

Metabolic network modelling wikipedia , lookup

Maternal physiological changes in pregnancy wikipedia , lookup

Transcript
Chapter 1
1. Voluntary micturition is initiated by inhibition of the:
A. Parasympathetic nerves.
B. Hypogastric sympathetic nerves.
C. Pudendal nerve.
D. Facial nerves.
E. Glossopharyngeal nerves.
2. For which compound the excretion can be higher than the filtered load?
A. Glucose.
B. Inulin.
C. Creatinine.
D. PAH.
E. None is correct.
3. A freely filterable substance that neither reabsorbed nor secreted has a renal artery
concentration of 12 mg/ml and a renal vein concentration of 9 mg/ml. Which of the following
is the filtration fraction (GFR/RPF)?
A. 0.05.
B. 0.15.
C. 0.25.
D. 0.35.
E. 0.45.
3. Assuming the GFR =100 ml/min and Tm = 375 mg/min, indicate the threshold
concentration of glucose in plasma (mg/ml) at which the glucose will appear in urine:
A. 8.75.
B. 5.75.
C. 3.75
D. 2.75
E. 1.75.
4. Laboratory tests have shown that the renal clearance of certain compound is 50 ml/min.
Indicate the specific mechanism of renal excretion.
A. Filtration.
B. Filtration + Reabsorption.
C. Secretion + Reabsorption.
D. Secretion + Filtration.
E. None is correct.
6. If a substance appears in the renal artery but not in the renal vein,
A. Its clearance is equal to GFR.
B. It must be reabsorbed by the kidney.
C. Its urinary concentration must be higher than its plasma concentration.
D. Its plasma clearance is equal to RPF.
E. It has the “filtration only” mechanism.
7. A previously well 12-year-old boy is brought to the emergency room with vomiting after a
prolonged period of exercise. Laboratory findings (high serum creatinine and urea) suggest
acute renal failure. Following treatment and recovery, his serum uric acid concentration (0.6
mg/dL) remains consistently below normal. What is the uric acid clearance if V = 1ml/min
and Uuric acid = 36 mg/dL.
A. 6 ml/min.
B. 12 ml/min.
C. 24 ml/min.
D. 48 ml/min.
E. 60 ml/min.
8. Use the following laboratory data to estimate the renal blood flow (RPF) by assuming that
PAH (p-aminohipuric acid) does not returns to systemic circulation:
Renal artery PAH = 0.15 mg/ml.
Urinary PAH = 19 mg/ml.
Urine flow = 5 ml/min.
RPF is approximately:
A. 800 ml/min.
B. 733 ml/min
C. 633 ml/min.
D. 590 ml/min.
E. 520 ml/min.
Chapter 2
1. A fall of PGC from 59 mm Hg to 49 mm Hg at afferent arteriole in dehydrated
individual will cause.
A. A drop of GFR to less then half.
B. A drop of GFR by 20%.
C. No change of creatinine in blood.
D. Two fold decrease of creatinine in blood.
E. No change in RBF.
2. Mark the incorrect answer. Vasodilation of the efferent arteriole will cause the
following effects:
A. GFR will decrease little bit.
B. RBF will increase.
C. Filtration fraction will stay constant.
D. Filtration fraction will decrease.
E. RPF will increase.
3. GFR and RBF will both be increased if:
A. Only the afferent arterioles is dilated.
B. The efferent and afferent arterioles are both constricted.
C. Only the afferent arteriole is constricted.
D. Only the efferent arteriole is constricted.
E. The afferent arteriole is constricted and the efferent is dilated.
4. Under certain patho-physiological conditions (e.g. hemorrhage) the decreased levels of
RBF may cause kidney ischemia. The release of which hormone may prevent the kidney
ischemia?
A. ADH.
B. ANP.
C. Renin.
D. Aldosterone.
E. PGI2 and PGE2.
5. Mark the incorrect answer. Macula Densa responds to the increased NaCl delivery into
the tubule by:
A. Releasing adenosine.
B. Constriction of the afferent arteriole of the same nephron.
C. Decreasing the GFR back to normal value.
D. Decreasing the RBF back to normal value.
E. Increasing both the GFR and RBF back to normal value.
6. A release of which hormone will cause a modest increase in GFR with little change in
RBF?
A. Adenosine.
B. ANP.
C. AngiotensineII.
D. Epinephrine.
E. Endotheline.
7. Mark the incorrect answer. A prolonged physical exercise of healthy person results
in increase of arterial blood pressure from 120 mm Hg to 170 mm Hg. This might be
associated with:
A. Vasoconstriction of afferent arterioles.
B. No significant change in filtered load of Na+.
C. Substantial increase in Na+ excretion.
D. No change in Na+ reabsorption.
E. No significant change in GFR and RBF.
8. The filtration fraction is increased by which of the following?
A. Increasing renal blood flow.
B. Increasing afferent arteriolar resistance.
C. Increasing efferent arteriolar resistance.
D. Increasing plasma oncotic pressure.
E. Increasing the pressure within the Bowman’s capsule.
Chapter 3
1. A 39-yer-old women is admitted to the hospital by her physician because of decreased
urine output. To distinguish the volume depletion from renal failure the sodium excretion
fraction must be estimated. The laboratory tests reveal:
Urine:
PNa = 135 mEq/L
PCr = 70 mg/L
Serum:
UNa= 40 mEq/L
UCr = 1000 mg/L
Her fractional sodium excretion is approximately:
A. 0.5%.
B. 1.0%.
C. 1.5%.
D. 2.0%.
E. 3.0%.
2. Mark the incorrect answer. Inhibition of the following systems may cause a diuretic
effect.
A. Loop Na+/K+/Cl- contransporter.
B. Distal tubule Na+/Cl- contransporter.
C. Proxmal tubular carbon anhydrase.
D. Collecting duct Ca2+-ATPase.
E. Collecting duct Na+-channel.
3. If GFR increases, proxymal tubular reabsorption of salt and water will increase by a
process called GT-balance. Contributions to this process includes which of the following?
A. An increase in peritubular capillary hydrostatic pressure.
B. A decrease in peritubular capillary sodium concentration.
C. An increase in peritubulary capillary oncotin pressure.
D. An increase of urine flow in proximal tubule.
E. An increase of urine flow in collecting duct.
4. What percentage of the filtered load of sodium is reabsorbed by the proximal tubule?
A. 20%
B. 25%.
C. 45%.
D. 65%.
E. 95%.
5. Mark the incorrect answer. Sodium reabsorption is coupled with:
A. Proton secretion.
B. Potassium secretion.
C. Bicarbonate reabsorption.
D. Amino acids secretion.
E. Glucose reabsorption.
6. Potassium-sparing diuretics inhibit sodium reabsorption in which of the following:
A. Proximal tubule.
B. The descending limb of Henle’s loop.
C. The ascending limb of Henle’s loop.
D. Collecting duct.
E. None is correct.
7. From the basolateral surface of the tubule the Na+ is reabsorbed into the intersticium
by which of the following:
A. Na/H exchange.
B. Na/glucose cotransport.
C. Na/K pump.
D. Na/amino acid cotransport.
E. Carbon anhydrase.
Chapter 5
1. A concurrent intake of Spironolactone (aldosterone antagonist) and ACE
(Angiontensin Converting Enzyme) inhibitor is dangerous because they may cause:
A.
B.
C.
D.
E.
Hypernatremia.
Hyperkalemia.
Elevated Na+-reabsorption.
Elevated K+-secretion.
Elevated K+-excretion.
2. Mark the incorrect answer. Proteinurea may occur in following cases:
A. Loss of negative charge on the filtration barrier.
B. Breakdown of blood red cells (hemolysis).
C. Breakdown of muscle cells (rhabdomyolysis).
D. Nephrogenic diabetic insidipus.
E. Impaired hydrolysis of proteins in proximal tubule.
3. Mark the incorrect answer. Hypoaldosteronism (low levels of aldosterone in plasma)
may cause the following effects:
A. Low tubular reabsorption of Na+.
B. Acidosis + Hyperkalemia.
C. Pulmonary edema.
D. Reduced ECV.
E. Reduced H+-secretion in the collecting duct.
4. The mechanism of glomerulotubular balance (GT-balance) is responsible for retaining
a constant:
A. Filtration fraction.
B. Na+ excretion (mEq/day).
C. Free water clearance (CH2O).
D. Plasma osmolality (Posm).
E. Urine osmolality (Uosm).
5. Decreased renal sympathetic activity will cause the following effects:
Na+ + H2O
excretion
GFR
ADH
Renin
Aldosterone
A.





B.





C.





D.





E.





6. A 23-year-old man is brought to the emergency room after collapsing during basketball
practice. On admission he appears confused. His coach reports that he was drinking a lot of
water during practice. His symptoms are most likely caused by increased
A. Intracellular tonicity.
B. Exracellular tonicity.
C. Hypernatremia.
D. Intracellular volume.
E. Hyperkanemia.
7. Renin release from the juxtaglomerular apparatus is inhibited by which of the following:
A. Beta-adrenergic agonists.
B. Prostaglandins.
C. Aldosterone.
D. Stimulation of Macula Densa.
E. Increased hydrostatic pressure within the afferent arterioles.
8. An individual is euvolemic and ingests a diet that contains 200 mEq/day
of sodium on average. What would be the sodium excretion rate of this individual over a 24hour period?
A. 400 mEq/day.
B. 200 mEq/day.
C. 120 mEq/day.
D. 100 mEq/day.
E. 50 mEq/day.
Chapter 6
1. The amount of potassium excreted by the kidney will decrease if
A. Distal tubular flow increases.
B. Circulating aldosterone levels increase.
C. Dietary intake of potassium increases.
D. Na+-reabsorption by the collecting duct decreases.
E. The excretion of organic ions decrease.
2. The extracellular potassium of a hyperkalemic patient can be decreased by
administering which of the following
A. Epinephrine.
B. Lactic acid.
C. Isotonic saline.
D. Hypertonic saline.
E. Loop diuretics.
3. A rapid reabsorption of K+ by gastrointestinal system (e.g., after a meat meal) may cause
dangerous levels of hyperkalemia. This can be prevented by immediate
A. Release of ADH in blood.
B. Release of Aldosterone in blood.
C. Release of Insulin in blood.
D. Elevation in H+-secretion.
E. Elevation in K+-secretion.
4. Which hormone inhibits the renal excretion of both the phosphate and calcium:
A. PTH.
B. Calcitriol.
C. Calcitonin.
D ADH.
E. Angiotensin II.
5. Hyperkalemia may be observed in patients with which of the following?
A. Volume depletion.
B. Diuretic therapy.
C. Administration of insulin.
D. Metabolic alkalosis.
E. ADH release.
6. Mark the incorrect answer. Parathyroid hormone (PTH) increases the plasma calcium
by
A. Stimulating bone resorption.
B. Increasing the calcium reabsorption by the kidney.
C. Stimulating the production of calcitriol.
D. Increasing (indirectly) the calcium reabsorption from GI-tract.
E. Decreasing the phosphate excretion.
7. Under normal physiological conditions the filtered load of which following is higher than
the Tm of reabsorption?
A. Phosphate.
B. Lactate.
C. Calcium.
D. Citrate.
E. Glucose.
8. 60-70% of filtered load of magnesium is reabsorbed in the
A. Proximal tubule.
B. Distal tubule.
C. Collecting duct.
D. Descending limb of Hanle’s loop.
E. Ascending limb of Hane’s loop.
Chapter 7
1. A 40-year-old woman is admitted to the intensive care unit with hypotension and
shortness of breath. Arterial blood gases reveal: PCO2= 10 mmHg and bicarbonate, 12
mEq/L. What is her acid-base status?
A. Normal.
B. Respiratory acidosis.
C. Metabolic acidosis.
D. Metabolic alkalosis.
E. Respiratory alkalosis.
2. Which one of the following statements about ammonia is correct?
A.
B.
C.
D.
E.
It is impermeable to the epithelial cells of the proximal tubule.
It is classified as a titratable acid.
It is produced by epithelial cells in the distal nephron.
It induces a bicarbonate concentration in the plasma.
Its synthesis is increased in respiratory acidosis.
3. A 29-year-old woman with a 3-day history of severe diarrhea comes to the emergency
room. She is hyperventilating and has postural hypotensen. Laboratory tests show:
Serum:
Arterial blood values:
[Na+] = 137 mEq/L
pH = 7.23
[Cl-] = 114 mEq/L
PCO2 = 26 mmHg
[HCO-3] = 13 mEq/L
The most likely diagnosis is:
A. Metabolic acidosis with abnormal anionic gap + partially respiratory compensation.
B. Metabolic acidosis with normal anionic gap + partially respiratory compensation.
C. Metabolic alkalosis + partially respiratory compensation.
D. Respiratory acidosis + partially metabolic compensation.
E. Respiratory alkalosis + partially metabolic compensation.
4. A 18-year-old girl is admitted to the hospital with a 4-day history of generalized muscle
cramps. Laboratory tests show:
Serum:
Arterial blood:
[Na+] = 149 mEq/L
pH = 7.50
[K+] = 2.0 mEq/L
[HCO-3] = 40 mEq/L
Which of the following is the most likely cause of this patient’s hypokalemic alkalemia?
A.
B.
C.
D.
E.
Hyperaldosteronism.
Hyperventilation.
Persistent diarrhea.
Renal failure.
Diabetes.
5. In which type of metabolic acidosis the ‘anionic gap’ is normal:
A. Diarrhea.
B. Renal failure.
C. Diabetic ketoacidosis.
D. Lactic acidosis.
E. Ingestion of acidic drug.
6. Under normal physiological conditions the rate of net
nonvolatile acid (mEq/day) produced in ECF must be equal to:
A. Filtered load of H+.
B. Tubular secretion of H+.
C. Net acid excretion (NAE).
D. Tubular reabsorption of HCO3-.
E. Excretion rate of HCO3-.
7. A respiratory acidosis that results in an increase in the concentration of hydrogen ions in
arterial blood from 40 nM (pH 7.4) to 50 nM (pH 7.3) would
A. Stimulate the peripheral and central chemo-receptors.
B. Decrease the amount of ammonium excreted in the urine.
C. Stimulate the osmo-receptors.
D. Increase the pH of the urine.
E. None is correct.
8. Use the following laboratory values to calculate the net acid excretion: Plasma pH=7.2,
Urine flow=1.2 L/day, Urine bicarbonate = 2 mEq/day, Urine titratable acids = 24 mEq/L,
Urine ammonium = 38 mEq/L, Urine pH = 5.4.
A. 55 mEq/L.
B. 64 mEq/L.
C. 68 mEq/L.
D. 72 mEq/L.
E. 76 mEq/L.